Difference between revisions of "2013 AMC 12B Problems/Problem 15"
Turkeybob777 (talk | contribs) m (→Problem) |
|||
Line 6: | Line 6: | ||
==Solution== | ==Solution== | ||
The prime factorization of <math> 2013 </math> is <math> 61*11*3 </math>. To have a factor of <math>61</math> in the numerator, <math>a_1</math> must equal <math>61</math>. Now we notice that there can be no prime <math>p</math> which is not a factor of 2013 such that <math> b_1<p<61</math> because this prime will not be represented in the denominator, but will be represented in the numerator. The highest <math> p </math> less than <math>61</math> is <math>59</math>, so there must be a factor of <math>59</math> in the denominator. It follows that <math>b_1 = 59</math>, so the answer is <math>|61-59|</math>, which is <math>\boxed{\textbf{(B) }2}</math>. One possible way to express <math> 2013 </math> is <cmath> \frac{61!*19!*11!}{59!*20!*10!}, </cmath> | The prime factorization of <math> 2013 </math> is <math> 61*11*3 </math>. To have a factor of <math>61</math> in the numerator, <math>a_1</math> must equal <math>61</math>. Now we notice that there can be no prime <math>p</math> which is not a factor of 2013 such that <math> b_1<p<61</math> because this prime will not be represented in the denominator, but will be represented in the numerator. The highest <math> p </math> less than <math>61</math> is <math>59</math>, so there must be a factor of <math>59</math> in the denominator. It follows that <math>b_1 = 59</math>, so the answer is <math>|61-59|</math>, which is <math>\boxed{\textbf{(B) }2}</math>. One possible way to express <math> 2013 </math> is <cmath> \frac{61!*19!*11!}{59!*20!*10!}, </cmath> | ||
+ | |||
+ | == See also == | ||
+ | {{AMC12 box|year=2013|ab=B|num-b=14|num-a=16}} |
Revision as of 18:06, 22 February 2013
Problem
The number is expressed in the form
where and are positive integers and is as small as possible. What is ?
Solution
The prime factorization of is . To have a factor of in the numerator, must equal . Now we notice that there can be no prime which is not a factor of 2013 such that because this prime will not be represented in the denominator, but will be represented in the numerator. The highest less than is , so there must be a factor of in the denominator. It follows that , so the answer is , which is . One possible way to express is
See also
2013 AMC 12B (Problems • Answer Key • Resources) | |
Preceded by Problem 14 |
Followed by Problem 16 |
1 • 2 • 3 • 4 • 5 • 6 • 7 • 8 • 9 • 10 • 11 • 12 • 13 • 14 • 15 • 16 • 17 • 18 • 19 • 20 • 21 • 22 • 23 • 24 • 25 | |
All AMC 12 Problems and Solutions |